LSAT and Law School Admissions Forum

Get expert LSAT preparation and law school admissions advice from PowerScore Test Preparation.

 Administrator
PowerScore Staff
  • PowerScore Staff
  • Posts: 8917
  • Joined: Feb 02, 2011
|
#31838
Please post below with any questions!
 medialaw111516
  • Posts: 80
  • Joined: Dec 11, 2018
|
#71868
Normally, I just look at the rules for the first question and eliminate the wrong answers, but this one was so hard for me to see or understand that I ended up eliminating C because it broke the most obviously rule about io on lower 4, but after that I was stumped and confused myself by drawing out the possibilities. Any clarification (especially with a drawn out diagram) would be super helpful.
 Jeremy Press
PowerScore Staff
  • PowerScore Staff
  • Posts: 1000
  • Joined: Jun 12, 2017
|
#71888
Hi medialaw,

This is a diagram that another instructor posted near the end of our setup thread, and I think it nicely illustrates the game.
Screen Shot 2019-11-11 at 12.13.23 PM.png
Note that the "double arrow" rules at the end of the diagram present the two possible arrangements of the four variables whose positioning we can't tell from the setup. You could have Fw with Ho (either one on top or bottom), and Go with Iw (again, either one on top or bottom). OR, you could have Fw with Go (either one on top or bottom), and Ho with Iw (either one on top or bottom).

For the first question to this game, you're right it's a tough one. The inferences are key to thinking through the answer choices, the biggest inference being that the upper position of Wall 4 must be Hw. That inference eliminates answer choices B and E. You can also eliminate answer choice D using the rules, because Franz's oil (Fo) must be in at least one of the lower positions, and it doesn't appear at all in the answer choice D list.

I hope this helps!

Jeremy
You do not have the required permissions to view the files attached to this post.
User avatar
 ariellelogan
  • Posts: 3
  • Joined: Aug 22, 2022
|
#97047
Can you walk me through how you made the inference that Hw has to be above Io?
 Robert Carroll
PowerScore Staff
  • PowerScore Staff
  • Posts: 1787
  • Joined: Dec 06, 2013
|
#97084
ariellelogan,

As is discussed in the setup thread, each wall must be one oil and one watercolor, and those must be by two different students. So above IO you must have a watercolor not by I. Further, the third rule means that watercolor cannot be by F either. G's watercolor is above F's oil, so the only watercolor that can be above IO is HW.

Robert Carroll

Get the most out of your LSAT Prep Plus subscription.

Analyze and track your performance with our Testing and Analytics Package.